Đến nội dung

Hình ảnh

Topic bất đẳng thức THCS (2)


  • Chủ đề bị khóa Chủ đề bị khóa
Chủ đề này có 1115 trả lời

#1081
pcfamily

pcfamily

    Thượng sĩ

  • Thành viên
  • 212 Bài viết

Cho $x\geq 1,y\geq 1,z\geq 1$
Chung Minh
$\frac{1}{1+x^{3}}+\frac{1}{1+y^{3}}+\frac{1}{1+z^{3}}\geq \frac{3}{1+xyz}$

Bđt 23
http://diendantoanho...hụ/page__st__20

#1082
ZzNarutozZ

ZzNarutozZ

    Binh nhất

  • Thành viên
  • 25 Bài viết
Với a,b>0 và a+b=1 cmr:
$(1+\frac{1}{a})(1+\frac{1}{b}) \geq 9$

Bài viết đã được chỉnh sửa nội dung bởi ZzNarutozZ: 11-12-2012 - 20:35


#1083
banhgaongonngon

banhgaongonngon

    Thượng úy

  • Thành viên
  • 1046 Bài viết

Với a,b>0 và a+b=1 cmr:
$(1+\frac{1}{a})(1+\frac{1}{b}) \geq 9$


Có $A=1+\frac{1}{a}+\frac{1}{b}+\frac{1}{ab} \geqslant 1+\frac{4}{a+b}+\frac{4}{(a+b)^{2}}=9$
Dấu "=" xảy ra khi $a=b=\frac{1}{2}>0$

#1084
banhgaongonngon

banhgaongonngon

    Thượng úy

  • Thành viên
  • 1046 Bài viết

Với a,b>0 và a+b=1 cmr:
$(1+\frac{1}{a})(1+\frac{1}{b}) \geq 9$


Hoặc bạn áp dụng luôn bất đẳng thức này nè $(1+x)(1+y)\geqslant (1+\sqrt{xy})^{2}$
Áp dụng : $(1+\frac{1}{a})(1+\frac{1}{b})\geqslant (1+\frac{1}{\sqrt{ab}})^{2}\geqslant (1+\frac{2}{a+b})^{2}=9$

Cách khác: Biến đổi tương đương: $(a+1)(b+1)\geqslant 9ab \Leftrightarrow ab+a+b+1\geqslant 9ab \Leftrightarrow ab\leqslant \frac{1}{4}$ (đúng theo $AM-GM$ )

Bài viết đã được chỉnh sửa nội dung bởi banhgaongonngon: 11-12-2012 - 20:58


#1085
Dung Dang Do

Dung Dang Do

    Dũng Dang Dở

  • Thành viên
  • 524 Bài viết

Với a,b>0 và a+b=1 cmr:
$(1+\frac{1}{a})(1+\frac{1}{b}) \geq 9$

ư
$(1+\frac{1}{a})(1+\frac{1}{b})=(1+\frac{1}{2a}+\frac{1}{2a})(1+\frac{1}{2b}+\frac{1}{2b})\ge 3\sqrt[3]{\frac{3}{4a^2}}*3\sqrt[3]{\frac{3}{4b^2}}\ge 9\sqrt[3]{\frac{9}{16abab}}\ge 9$
__________
P/s: Mấy cách dễ thì các bạn làm hết rồi. Thôi thì cách này tạm vậy
@@@@@@@@@@@@

#1086
nguyencuong123

nguyencuong123

    Thiếu úy

  • Thành viên
  • 587 Bài viết
Cho x,y,z la cac so thuc duong. CM

$\frac{1}{x\left ( y+1 \right )}+\frac{1}{y\left ( z+1 \right )}+\frac{1}{z\left ( x+1 \right )}\geq \frac{3}{xyz+1}$
____________
Nhập số bài vào lần sau sẽ xoá

Bài viết đã được chỉnh sửa nội dung bởi Dung Dang Do: 13-12-2012 - 17:34

    :icon12:  :icon12:  :icon12:   Bình minh tắt nắng trời vương vấn :icon12:  :icon12:  :icon12:       

      :icon12: Một cõi chơi vơi, ta với ta  :icon12:       

:nav: My Facebook  :nav:  

 


#1087
Joker9999

Joker9999

    Thiếu úy

  • Thành viên
  • 659 Bài viết

Cho x,y,z la cac so thuc duong. CM

$\frac{1}{x\left ( y+1 \right )}+\frac{1}{y\left ( z+1 \right )}+\frac{1}{z\left ( x+1 \right )}\geq \frac{3}{xyz+1}$
____________
Nhập số bài vào lần sau sẽ xoá

Tham khảo tại đây:
http://diendantoanho...1geq-frac31abc/

<span style="font-family: trebuchet ms" ,="" helvetica,="" sans-serif'="">Nỗ lực chưa đủ để thành công.


.if i sad, i do Inequality to become happy. when i happy, i do Inequality to keep happy.

#1088
Kwon Simonster

Kwon Simonster

    Binh nhì

  • Thành viên
  • 13 Bài viết

Cho x,y,z la cac so thuc duong. CM

$\frac{1}{x\left ( y+1 \right )}+\frac{1}{y\left ( z+1 \right )}+\frac{1}{z\left ( x+1 \right )}\geq \frac{3}{xyz+1}$
____________
Nhập số bài vào lần sau sẽ xoá


Đề: C/m: $\frac{1}{a(b+1)}+\frac{1}{b(c+1)}+\frac{1}{c(a+1)}\geq \frac{3}{abc+1}$

Đặt $a = k\frac{x}{y} , b=k\frac{y}{z},c=k\frac{z}{x} => abc=k^{3}$
Từ giả thiết => cần chứng minh $\frac{1}{k^{2}\frac{x}{z}+k\frac{x}{y}}+\frac{1}{k^{2}\frac{y}{x}+k\frac{y}{z}}+\frac{1}{k^{2}\frac{z}{y}+k\frac{z}{x}}\geq \frac{3}{k^{3}+1}$

Ta xét :
$VT=\frac{yz}{k^{2}xy+kxz}+\frac{xz}{k^{2}yz+kxy}+\frac{xy}{k^{2}zx+kyz} =\frac{(yz)^{2}}{k^{2}(xy)(yz)+k(xz)(yz)}+\frac{(xz)^{2}}{k^{2}(yz)(xz)+k(xy)(xz)}+\frac{(xy)^{2}}{k^{2}(xy)(xz)+k(yz)(yx)}\geq \frac{(yz+xz+xy)^{2}}{(k^{2}+k)(xy^{2}z+x^{2}yz+xyz^{2})}\geq \frac{3}{k^{2}+k}\geq \frac{3}{k^{3}+1}$

Áp dụng bất đặng thức sau để chứng minh: $\frac{a^{2}}{x}+\frac{b^{2}}{y}+\frac{c^{2}}{z}\geq \frac{(a+b+c)^{2}}{x+y+z}$
Dễ dàng chứng minh được: $k^{2}+k\leq k^{3}+1\Leftrightarrow k(k+1)\leq (k+1)(k^{2}-k+1)$
Luôn đúng với mọi k >0

--------------------------------------
P/s: Mình xin lỗi vì lỡ đánh nhầm nên đành sửa x, y , z thành a, b, c nhé bạn

Bài viết đã được chỉnh sửa nội dung bởi Kwon Simonster: 16-12-2012 - 21:38

Hình đã gửi


Simonster Love SNSD

Girl's Generation We don't stop


#1089
Supermath98

Supermath98

    Thiếu úy

  • Thành viên
  • 512 Bài viết

Trong toppic này có mấy kí hiệu em không hiểu. mọi người nói cho em biết khi nào thì dùng nó cái. kí hiệu $\sum$


:icon12: :icon12: :icon12: Đừng bao giờ ngồi một chỗ và ước. Hãy đứng dậy và làm:icon12: :icon12: :icon12:

#1090
eatchuoi19999

eatchuoi19999

    Sĩ quan

  • Thành viên
  • 320 Bài viết

Mọi người giúp em bài này với. Cho $a,b,c> 0; a+b+c=3$. Chứng minh:

$\frac{a^{3}}{b^{2}+c^{2}}+\frac{b^{3}}{c^{2}+a^{2}}+\frac{c^{3}}{a^{2}+b^{2}}\geqslant \frac{3}{2}$



#1091
duybigbangvip

duybigbangvip

    Binh nhì

  • Thành viên
  • 11 Bài viết

tìm giá trị lớn nhất và nhỏ nhất của :

$\tfrac{x^2-(x-4y)^2}{x^2+4y^2}$



#1092
nguyensidang

nguyensidang

    Binh nhất

  • Thành viên
  • 32 Bài viết

Trong toppic này có mấy kí hiệu em không hiểu. mọi người nói cho em biết khi nào thì dùng nó cái. kí hiệu $\sum$

đấy là kí hiệu tổng bạn ạ

vd:$\sum \frac{a}{b+c}$

ta sẽ hiểu là 1 tổng với các số hạng là hoán vị các biến của biểu thức gốc

$\frac{a}{b+c}+\frac{b}{c+a}+\frac{c}{a+b}$



#1093
andymurray44

andymurray44

    Trung sĩ

  • Thành viên
  • 153 Bài viết

Mọi người giúp em bài này với. Cho $a,b,c> 0; a+b+c=3$. Chứng minh:

$\frac{a^{3}}{b^{2}+c^{2}}+\frac{b^{3}}{c^{2}+a^{2}}+\frac{c^{3}}{a^{2}+b^{2}}\geqslant \frac{3}{2}$

Bài này đc phết!

Bước 1

$\frac{2a^{3}}{b^{2}+c^{2}}+a\geq \frac{2\sqrt{2}a^{2}}{\sqrt{b^{2}+c^{2}}}$

Tương tự,ta có thể xử lí đc phần cộng thêm là a,b,c vì chúng có tổng bằng 3.Bây giờ chỉ cần tìm min biểu thức

$\sum \frac{a^{2}}{\sqrt{b^{2}+c^{_{2}}}}$

Áp dụng BĐT Cauchy:

$\sum \frac{a^{2}}{\sqrt{b^{2}+c^{_{2}}}}\geq \sum \frac{2a^{2}}{b^{2}+c^{_{2}}}$

Giờ tìm min của$\sum \frac{2a^{2}}{b^{2}+c^{_{2}}}$

Cái này thì dễ rùi,cộng 2 vào mỗi phân số rồi áp dụng BĐT $\frac{1}{x}+\frac{1}{y}+\frac{1}{z}\geq \frac{9}{x+y+z}$ xong luôn,đó là cơ bản hướng làm.Mình lười nên ko muốn tính hẳn số rõ ràng ra nhưng chắc là đúng thui :icon6:


Bài viết đã được chỉnh sửa nội dung bởi andymurray44: 31-03-2013 - 16:15


#1094
andymurray44

andymurray44

    Trung sĩ

  • Thành viên
  • 153 Bài viết


Bài viết đã được chỉnh sửa nội dung bởi andymurray44: 31-03-2013 - 16:17


#1095
Trang Luong

Trang Luong

    Đại úy

  • Thành viên
  • 1834 Bài viết

Cho a,b,c > 0 CMR :

$\sum \frac{bc}{a^{2}(b+c)}\geq \frac{1}{2}\left ( \sum \frac{1}{a} \right )$


"Nếu bạn hỏi một người giỏi trượt băng làm sao để thành công, anh ta sẽ nói với bạn: ngã, đứng dậy là thành công"
Issac Newton

#1096
Kwon Simonster

Kwon Simonster

    Binh nhì

  • Thành viên
  • 13 Bài viết

Chứng minh rằng

 

$\frac{1}{3(1+\sqrt{2})}+\frac{1}{5(\sqrt{2}+\sqrt{3})}+\frac{1}{7(\sqrt{3}+\sqrt{4})}+...+\frac{1}{4015(\sqrt{2007}+\sqrt{2008})}< \frac{2007}{2009}$

 

Mình cần giải bài này gấp. mong mọi người giúp đỡ 

Cảm ơn nhìu ạ =))


Bài viết đã được chỉnh sửa nội dung bởi Kwon Simonster: 01-04-2013 - 21:14

Hình đã gửi


Simonster Love SNSD

Girl's Generation We don't stop


#1097
eatchuoi19999

eatchuoi19999

    Sĩ quan

  • Thành viên
  • 320 Bài viết

Cho $x,y> 0$ thỏa mãn $xy\geqslant 2$. Chứng minh:

$(x-2)^{2}+(y+2)^{2}\geqslant 8$



#1098
Trang Luong

Trang Luong

    Đại úy

  • Thành viên
  • 1834 Bài viết

Cho A = $\frac{1}{1^{2}+2^{2}+3^{2}}+\frac{1}{2^{2}+3^{2}+4^{2}}+...+\frac{1}{24^{2}+25^{2}+26^{2}}$

CMR : $0,15 < A < 0,25$


"Nếu bạn hỏi một người giỏi trượt băng làm sao để thành công, anh ta sẽ nói với bạn: ngã, đứng dậy là thành công"
Issac Newton

#1099
Trang Luong

Trang Luong

    Đại úy

  • Thành viên
  • 1834 Bài viết

Cho $x>0, y>0$ thỏa $x+y=\frac{6}{5}$. Tìm GTNN của $S=\frac{5}{x}+\frac{1}{5y}$


"Nếu bạn hỏi một người giỏi trượt băng làm sao để thành công, anh ta sẽ nói với bạn: ngã, đứng dậy là thành công"
Issac Newton

#1100
4869msnssk

4869msnssk

    Bá tước

  • Thành viên
  • 549 Bài viết

cho a,b,c là các số dương, chứng minh rằng: $\frac{1}{a}+\frac{2}{b}+\frac{3}{c}\geq \frac{3}{a+b}+\frac{18}{3b+4c}+\frac{9}{c+6a}$


 B.F.H.Stone





3 người đang xem chủ đề

0 thành viên, 3 khách, 0 thành viên ẩn danh